• Không có kết quả nào được tìm thấy

Ước chung lớn nhất và Bội chung nhỏ nhất - Phạm Thị Thu Hiền

N/A
N/A
Protected

Academic year: 2022

Chia sẻ "Ước chung lớn nhất và Bội chung nhỏ nhất - Phạm Thị Thu Hiền"

Copied!
23
0
0

Loading.... (view fulltext now)

Văn bản

(1)

ƯỚC CHUNG LỚN NHẤT, BỘI CHUNG NHỎ NHẤT VÀ LIÊN HỆ GIỮA CHÚNG

PHẠM THỊ THU HIỀN Chuyên Hùng Vương - Phú Thọ Email: Phamhienchv@gmail.com Các khái niệm ước số chung lớn nhất (GCD) và bội số chung nhỏ nhất (LCM) là những khái niệm cơ bản của số học. Cách đây khoảng 2000 năm, con người đã biết đến thuật toán Euclid nổi tiếng và có lẽ là thuật toán cổ nhất của toán học để tìm ước số chung lớn nhất của hai số nguyên a và b với b 6= 0. Với mục đích tìm hiểu sâu hơn sự vận dụng của hai khái niệm này, báo cáo tập trung trình bày một số tính chất của ước số chung lớn nhất, bội số chung nhỏ nhất và mối liên hệ giữa chúng cùng các ví dụ điển hình được chọn lọc từ các cuộc thi chọn học sinh giỏi của các nước.

1 Ước chung lớn nhất

Định nghĩa 1.1. Cho hai số nguyêna và b trong đó ít nhất một số khác 0 . Số nguyên dương d được gọi là ước số chung lớn nhất của a và b nếu:

(i) d | a và d | b (d là ước số chung của a và b), (ii) Nếu c | a và c | b thì c | d.

Kí hiệu ước số chung lớn nhất của a và b là gcd (a, b) hoặc (a, b). Nếu gcd (a, b) = 1 thì ta nói a và b là hai số nguyên tố cùng nhau.

Tính chất 1.2. Sau đây là một số tính chất của ước số chung lớn nhất

(i) gcd (a, b) =d ⇔ gcd a

d; b d

= 1.

(ii) Choclà một số nguyên dương. Khi đó ta cógcd (ca, cb) =c.gcd (a, b). (iii) gcd (a, b) = 1 và b | ac thì b | c.

(2)

(iv) Nếu gcd (a, b) = 1 và gcd (a, c) = 1 thì gcd (a, bc) = 1.

(v) Nếu a = bq +r thì gcd (a, b) = gcd (b, r).

(vi) Nếu a = pα11.pα22...pαkk và b = pβ11.pβ22...pβkk với αi, βi ∈ N, αii ≥1, i = 1,2, ..., k, thì gcd (a, b) = pmin1 {α11}.pmin2 {α22}...pmink {αkk}.

Khái niệm ước số chung lớn nhất được mở rộng tự nhiên cho trường hợp có hơn hai số theo cách như sau

gcd (a1, a2, ..., an1, an) = gcd (gcd (a1, a2, ..., an1), an).

Định lí 1.3 (Bachet - Bezout). Với hai số nguyên dương a và b, luôn tồn tại các số nguyên x và y sao cho ax+ by = gcd (a, b).

Định lí 1.4. Với các số nguyên a, b ta có a2, b2

= (a, b)2.

Chứng minh. Trước hết ta chứng minh bổ đề: Cho các số nguyên a, b, c khác 0. Khi đó ta có

(a, bc) = (a,(a, b)c).

Ta có (a,(a, b)c) | ((a, b)c) | bc. Do đó (a,(a, b)c) là ước của a và bc, suy ra (a,(a, b)c) | (a, bc).

Mặt khác, (a, bc) là ước của a và bc nên nó là ước của ac và bc. Vì vậy (a, bc) | (ac, bc) = c(a, b). Ta có (a, bc) là ước của a và c(a, b) nên (a, bc) | (a,(a, b)c).

Vậy (a, bc) = (a,(a, b)c).

Giả sử m, n là hai số nguyên tố cùng nhau, tức là (m, n) = 1. Áp dụng bổ đề vừa chứng minh ta có

m2, n2

= m2, m2, n n

= m2,(n,(m, n)m)n . Vì (m, n) = 1 nên m2,(n,(m, n)m)n

= m2, n

. Lại áp dụng bổ đề ta có

m2, n

= (n,(m, n)m) = 1.

Vì vậy (m, n) = 1 suy ra m2, n2

= 1.

Ta có theo tính chất 1.2. (i) a

(a, b), b (a, b)

= 1

(3)

suy ra

a2

(a, b)2, b2 (a, b)2

= 1.

Từ đó theo tính chất 1.2. (ii) ta có a2, b2

= (a, b)2.

Ví dụ 1.5. Cho hai số nguyên a, b nguyên tố cùng nhau. Chứng minh rằng

a+ b, a2 −ab+b2

= 1 hoặc 3.

Lời giải. Giả sử d = a+b, a2 −ab+b2

. Ta có d | (a+b)2 −a2 +ab−b2 = 3ab.

Do đó d | 3b(a+b)−3ab = 3b2. Tương tự d | 3a2. Do đó d | 3a2,3b2

= 3 a2, b2

= 3 (a, b)2 = 3.

Từ đó ta có điều phải chứng minh.

Ví dụ 1.6 (1997 Russian Mathematical Olympiad). Tìm tất cả các bộ ba số tự nhiên m, n, l thỏa mãn đồng thời các điều kiện sau

m+n = (m, n)2, m+l = (m, l)2, n+l = (n, l)2.

Lời giải. Đặt d = gcd (m, n, l). Khi đó ta có m = dm1, n = dn1, l = dl1 với gcd (m1, n1, l1) = 1.

Đặt dmn = gcd (m1, n1). Theo giả thiết ta có d(m1 +n1) = d2d2mn. Suy ra m1 +n1 = dd2mn. Tương tự như vậy với dnl và dlm ta có

2 (m1 +n1 + l1) = d d2mn +d2nl +d2lm . Ta có d

gcd (d,2) là ước của m1 +n1+l1 và cũng là ước của m1 +n1 nên d

gcd (d,2) là ước của l1. Tương tự ta có nó cũng là ước của m1 và n1. Vì gcd (m1, n1, l1) = 1 nên d

gcd (d,2) = 1, và do đó d ≤2.

Mặt khác dmn, dnl, dlm đôi một nguyên tố cùng nhau, suy ra l1 = l2dlmdln, m1 = m2dlmdmn, n1 = n2dnmdnl

(4)

và ta có

m2dlm +n2dln = ddmn.

Không làm mất tổng quát ta giả sử dmn = min{dmn, dlm, dln} ta có 2dmn ≥ ddmn = m2dlm+n2dln ≥ dlm+ dln ≥ 2dmn.

Từ bất đẳng thức trên suy ra d = 2, m2 = n2 = 1 và dmn = dlm = dln mà các số này nguyên tố cùng nhau nên chúng bằng 1. Từ đó suy ra m1 = n1 = 1 và từ l1 +m1 = dd2lm có l1 = 1.

Vậy m = n= l = 2.

Ví dụ 1.7. Cho dãy số Fibonacci F1 = F2 = 1, Fn+2 = Fn+1 +Fn, với n≥ 1. Chứng minh rằng

(i) gcd (Fn+1, Fn) = 1,

(ii) Nếu s, t là các số nguyên s ≥1, t ≥ 0 thì Fs+t = Fs1Ft +FsFt+1, (iii) Nếu n| m thì Fn | Fm,

(iv) gcd (Fm, Fn) = Fgcd(m,n).

Lời giải. (i) Đặt d = (Fn, Fn+1). Ta có d | Fn và d | Fn+1 nên d | Fn+1−Fn = Fn1. Do đó d | Fn −Fn1 = Fn2. Tiếp tục quá trình như vậy ta suy ra d | F1 = 1. Vì vậy ta có d = 1.

(ii) Ta cố định t và chứng minh quy nạp theo s.

Với s = 1 ta có Ft+1 = F0Ft +F1Ft+1 là hiển nhiên.

Giả sử s > 1 và Fsk+t = Fsk1Ft + FskFt+1 với mọi k thỏa mãn 1≤ k ≤s−1.

Khi đó ta có

Fs+t = Fs+t1 + Fs+t2

= Fs1+t+ Fs2+t

= Fs2Ft +Fs1Ft+1+ Fs3Ft+ Fs2Ft+1

= Ft(Fs2 +Fs3) + Ft+1(Fs1 +Fs2)

= FtFs1 +Ft+1Fs

(5)

Đến đây ta kết thúc chứng minh.

(iii) Cố định n. Ta chứng minh quy nạp theo k.

Nếu k = 1 thì rõ ràng Fn | Fn.

Giả sử Fn | Fkn. Ta đi chứng minh Fn | F(k+1)n. Thật vậy, đặt s = n, t= kn vào đẳng thức (ii) ta có

F(k+1)n = Fn+kn = Fn1Fkn +FnFkn+1. Từ đó suy ra Fn | F(k+1)n.

(iv) Đặt d = (Fn, Fm) và a = (m, n). Ta đi chứng minh d| Fa và Fa | d.

Thật vậy, vì a | m và a | n nên Fa | Fm, Fa | Fn suy ra Fa | d.

Nếu d = 1 thì hiển nhiên d | Fa. Trong trường hợp d > 1. Theo định lí Bezout, tồn tại hai số nguyên x, y sao cho xm +yn = a. Hiển nhiên không thể xảy ra trường hợp cả x và y là nguyên âm. Vì a | m, a | n nên a ≤ m, a ≤ n. x, y cũng không thể cùng là các số nguyên dương vì nếu như vậy thì a = xm+ yn ≥ m +n, mâu thuẫn. Do đó, x và y trái dấu nhau, không mất tổng quát ta giả sử x ≤ 0, y > 0.

Ta có Fyn = Faxm = Fa1Fxm+FaFxm+1.

Ta cón | ny, m | (−xm), do đóFn |Fyn,Fm | Fxm. Từ đó suy ra d | Fyn và d | Fxm. Vì thế d | FaFxm+1.

Dễ dàng thấy được d | Fm | Fxm. Nếu (d, Fxm+1) = k > 1 thì k | Fxm+1 và k | Fxm. Mà hai số Fibonacci liên tiếp nguyên tố cùng nhau.

Ta có mâu thuẫn.

Từ đó (d, Fxm+1) = 1, suy ra d | Fa. Đến đây ta hoàn thành chứng minh.

Ví dụ 1.8. Chứng minh rằng không tồn tại số Fibonacci lẻ nào chia hết cho 17.

Lời giải. Đặt d = (17, Fn), hiển nhiên d là số nguyên dương lẻ. Ta có (17, Fn) = (34, Fn) = (F9, Fn) = F(9,n) = F1, F3 hoặc F9. Điều đó có nghĩa d = 1,2 hoặc 34. Vì d lẻ nên ta có d = 1. Từ đó ta có điều phải chứng minh.

Ví dụ 1.9. Xét các số Fermat fn = 22n + 1. Chứng minh rằng với các số nguyên dương m, n, m > n ta có (fm, fn) = 1.

(6)

Lời giải. Ta có với mọi số nguyên dương m fm −2 = 22m−1

= 22n1.2 −1

= (fm1 −1)2 −1

= fm21 −2fm1

= fm1(fm1 −2)

= fm1fm2(fm2 −2)

= · · ·

= fm1fm2· · ·f1f0.

Giả sử d = (fm, fn) thì d | fn | fm−2. Do đó d| [fm−(fm −2)] = 2.

Rõ ràng d là số nguyên dương lẻ. Vì vậy fm và fn nguyên tố cùng nhau.

Ví dụ 1.10. Chof (x) = x3−x+1. Kí hiệuan(x) =f (f (...(f (x))...)), nlần f. Chứng minh rằng các số trong dãy m, a1(m), a2(m), ... đôi một nguyên tố cùng nhau với mọi số tự nhiên m > 1.

Lời giải. Ta có f (x)−1 = x3 −x = (x−1)x(x+ 1). Đặt ai = ai(m).

Khi đó ta có

an+k −1 = (an+k1 −1)an+k1(an+k1 + 1)

= (an+k2 −1)an+k2(an+k2 + 1)an+k1(an+k1+ 1)

= · · ·

= (an −1)anan+1· · ·ank+1(an + 1) (an+1 + 1)· · ·(an+k1 + 1). Do đó với các số tự nhiên i, j và i > j > 1 ta có aj | (ai−1). Giả sử d = (ai, aj) thì d | [ai −(ai −1)] = 1. Do đó d = 1. Vậy các số trong dãy m, a1(m), a2(m), ... đôi một nguyên tố cùng nhau với mọi số tự nhiên m > 1.

Ví dụ 1.11. Cho f (x) là một đa thức với các hệ số nguyên. Kí hiệu a0 = 0,an = f (an1)với mọin≥ 1. Chứng minh rằng(am, an) =|a(m,n)|. Khi f (x) = P9

i=2

xi+ 2, hãy tìm (a2016, a2014).

(7)

Lời giải. Đẳng thức

(am, an) = |a(m,n)| (1.1)

hiển nhiên đúng khi m = 0 hoặc n= 0. Bây giờ ta xét m, n > 0.

Để chứng minh đẳng thức (1.1) trước hết ta chứng minh tính chất

anq...an (1.2)

đúng với mọi n, q ∈ N.

Chứng minh tính chất (1.2) bằng quy nạp theo q.

Hiển nhiên (1.2) đúng khi q = 1.

Giả sử (1.2) đúng với q −1 tức là ta có an(q1)...an. Ta cần chứng minh anq...an.

Với mỗi số nguyên dương k ta đặt fk(x) =f (f (...(f (x))...)), k lần f. Ta có

anq −an = fn an(q1)

−fn(0)...an(q1). Kết hợp với giả thiết quy nạp, ta có (1.2) đúng với q.

Bây giờ để chứng minh (1.1), tương tự như thuật toán Euclid, ta chỉ ra rằng, nếu m = nq +r thì

(am, an) = (an, ar) (1.3) Thật vậy ta có

am−ar = fr(anq)−fr(0)...anq. Kết hợp với tính chất (1.2) ta có

am −ar...an

Từ đó ta có đẳng thức (1.3) đúng.

Khi f (x) = P9

i=2

xi+ 2 ta có

(a2016, a2014) =a2 = 1024.

Ví dụ 1.12. Cho n là một số tự nhiên. Tìm ước chung lớn nhất của 2n

1

, 2n

3

, ...,

2n 2n−1

.

(8)

Lời giải. Ta có

n

X

k=1

2n 2k−1

= 22n1,

suy ra ước chung lớn nhất cần tìm phải có dạng 2a. Ta giả sử n = 2lm với m là số tự nhiên lẻ. Mặt khác, ước chung lớn nhất của các số trên là ước của

2n 1

= 2n nên nó nhỏ hơn hoặc bằng 2l+1. Ta đi chứng minh 2l+1 là ước số chung lớn nhất cần tìm. Ta có

2l+1m 2k −1

= 2l+1m 2k −1

2l+1m−1 2k −2

.

Nhưng 2k −1∤ 2l+1 với k > 1 nên 2l+1 |

2l+1m 2k −1

. Từ đó ước chung lớn nhất của các số đã cho bằng 2l+1.

2 Bội chung nhỏ nhất

Định nghĩa 2.1. Cho hai số nguyên a và b khác 0. Số nguyên dương k được gọi là bội số chung nhỏ nhất của a và b nếu:

(i) a | k, b | k (k là bội số chung của a và b), (ii) Nếu a | t và b | t thì k | t.

Kí hiệu bội số chung nhỏ nhất của a và b là lcm (a, b) hoặc [a, b].

Tính chất 2.2. Sau đây là một số tính chất của bội số chung nhỏ nhất (i) Nếu lcm (a, b) = k thì gcd

k a,k

b

= 1.

(ii) t.lcm (a, b) = lcm (ta, tb).

(iii) Nếu d| a, d | b và d > 0 thì lcm a

d, b d

= lcm (a, b)

d .

(iv) Nếu a = pα11.pα22...pαkk và b = pβ11.pβ22...pβkk với αi, βi ∈ N, αii ≥1, i = 1,2, ..., k, thì lcm (a, b) = pmax1 {α11}.pmax2 {α22}...pmaxk {αkk}.

(9)

Khái niệm bội số chung nhỏ nhất được mở rộng tự nhiên cho trường hợp có hơn hai số theo cách như sau

lcm (a1, a2, ..., an1, an) = lcm (lcm (a1, a2, ..., an1), an). Ví dụ 2.3. Chứng minh rằng

lcm (1,2, ...,200) = lcm (101,102, ...,200).

Lời giải. Ta đi chứng minh bài toán tổng quát: Với mọi số nguyên dương n ta có

lcm (1,2, ...,2n) = lcm (n+ 1, n+ 2, ...,2n).

Đặt A = lcm (1,2, ...,2n), B = lcm (n+ 1, n+ 2, ...,2n). Hiển nhiên ta có A...B.

Bây giờ ta sẽ chứng minh B...A. Ta đi chứng minh B...k với mọi k = 1,2, ...,2n.

Nếu k ∈ {n+ 1, n+ 2, ...,2n} thì hiển nhiên B...k.

Nếu k ∈ {1,2, ..., n} thì trong k số nguyên dương liên tiếp n + 1, n+ 2, ..., n+k có một số chia hết cho k. Giả sử đó là a. Ta có

n+ 1 ≤a ≤ n+k ≤ 2n, do đó B...k.

Tóm lại, với mọi k ∈ {1,2, ...,2n} ta có B...k. Vì vậy B...A.

Vậy A = B.

Ví dụ 2.4. Hỏi có bao nhiêu bộ ba số nguyên dương (a, b, c) thỏa mãn lcm (a, b) = 1000, lcm (b, c) = 2000, lcm (c, a) = 2000.

Lời giải. Ta thấy cả hai số 1000 và 2000 đều có dạng 2m5n nên các số a, b, c đều phải có dạng này. Đặt a = 2m15n1, b = 2m25n2, c = 2m35n3, trong đó mi, ni là các số nguyên không âm với i = 1,2,3.

Từ giả thiết ta có

max{m1, m2} = 3, max{m2, m3}= 4, max{m3, m1} = 4 (2.1) và

max{n1, n2}= 3, max{n2, n3} = 3, max{n3, n1} = 3. (2.2)

(10)

Từ (2.1) ta có m3 = 4, hai số m1, m2 có một số bằng 3 và một số nhận một trong các giá trị 0,1,2,3. Có 7 bộ (m1, m2, m3) thỏa mãn đó là (0,3,4), (1,3,4), (2,3,4), (3,0,4), (3,1,4), (3,2,4) và (3,3,4).

Từ (2.2) ta có hai trong ba số n1, n2, n3 bằng 3 và số còn lại nhận một trong các giá trị 0,1,2,3. Ta có10 bộ(n1, n2, n3) thỏa mãn đó là(3,3,0), (3,3,1), (3,3,2), (3,0,3), (3,1,3), (3,2,3), (0,3,3), (1,3,3), (2,3,3) và (3,3,3).

Như vậy ta có tất cả 7.10 = 70 bộ số thỏa mãn đề bài.

Ví dụ 2.5 (Putnam 1980). Hỏi có bao nhiêu bộ 4số (a, b, c, d)thỏa mãn 3r7s = [a, b, c] = [b, c, d] = [c, d, a] = [d, a, b].

Lời giải.Vì sự phân tích thành các nhân tử nguyên tố của một số nguyên là duy nhất nên mỗi số a, b, c, d đều phải có dạng 3mi7ni với 0≤ mi ≤ r, 0≤ ni ≤s, i = 1,2,3,4.

Hơn nữa, mi bằng r đối với ít nhất hai số trong 4 số mi. Tương tự ni bằngs đối với ít nhất hai trong4 số ni. Ta có

4 2

r2 = 6r2 cách chọn hai trong bốn số mi bằng r và hai số mũ mi còn lại nhận các giá trị trong tập {0,1, ..., r−1}, có

4 3

r = 4r cách chọn ba trong bốn số mi bằng r và một số mi còn lại nhận giá trị trong tập {0,1, ..., r−1}, có 4

4

= 1 cách chọn cả bốn số mi bằng r. Như vậy ta có 1 + 4r + 6r2 cách chọn ít nhất hai trong bốn số a, b, c, d có lũy thừa của 3 bằng r.

Tương tự như vậy ta có 1 + 4s+ 6s2 cách chọn ít nhất hai trong bốn số a, b, c, d có lũy thừa của 7 bằng s. Như vậy ta có

1 + 4r + 6r2

1 + 4s+ 6s2 bộ bốn số thỏa mãn đề bài.

Ví dụ 2.6 (AMM E2686). Cho số nguyên dương n. Chứng minh rằng (n+ 1) lcm

n 0

,

n 1

, ..., n

n

= lcm (1,2, ..., n+ 1). (2.3)

(11)

Lời giải. Trước hết ta nhắc lại định lí: Giả sử n là số nguyên dương.

Khi đó lũy thừa của số nguyên tố p xuất hiện trong phân tích ra thừa số nguyên tố của n! là

n p

+

n p2

+

n p3

+· · ·

Giả sử p là một số nguyên tố, p ≤ n+ 1 và α (tương ứng β) là số mũ lớn nhất của p ở vế trái (tương ứng vế phải) của (2.3). Chọn số r thỏa mãn pr ≤ n+ 1 < pr+1. Rõ ràng β = r.

Ta cần chứng minh rằng, nếu pr ≤ m < pr+1 thì pr+1 ∤ m

k

với 0≤ k ≤m (*).

Thật vậy, lũy thừa của p xuất hiện trong sự phân tích ra thừa số nguyên tố của

m k

γ =

r

X

s=1

m ps

− k

ps

m−k ps

.

Mỗi hạng tử trong tổng trên bằng 0hoặc 1, ta có γ ≤ r, do đó (*) đúng.

Với 0 ≤ k ≤ n, đặt ak = (n+ 1)

n k

= (n−k+ 1)

n+ 1 k

= (k+ 1)

n+ 1 k + 1

.

Từ (*) ta có pr+1 không là ước của các số nguyên n k

,

n+ 1 k

, n+ 1

k + 1

. Vì vậy pr+1 là ước của ak khi và chỉ khi p là ước của cả ba số n+ 1, n−k + 1, k + 1. Từ đó suy ra

p | [(n+ 1)−(n−k+ 1)−(k + 1)] = −1, mâu thuẫn. Vì vậy pr+1 ∤ ak.

Mặt khác với k = pr −1 ta có k ≤ n nên ak = (k+ 1)

n+ 1 k+ 1

chia hết cho pr.

Vậy β = r = α. Từ đó suy ra điều phải chứng minh.

(12)

Ví dụ 2.7 (APMO 1998). Tìm số nguyên n lớn nhất thỏa mãn n chia hết cho tất cả các số nguyên nhỏ hơn √3 n.

Lời giải. Ta thấy 7 < √3

420 < 8 và 420 = lcm{1,2,3,4,5,6,7}. Ta đi chứng minh 420 là số cần tìm.

Giả sử n > 420 là số nguyên thỏa mãn chia hết cho tất cả các số nguyên nhỏ hơn√3 n. Ta có√3

n > 7nênnchia hết cho420 = lcm{1,2,3,4,5,6,7}, do đón≥ 840và√3

n > 9. Do đónchia hết2520 = lcm{1,2,3,4,5,6,7,8,9} và ta lại có √3

n > 13. Giả sử m là số nguyên dương lớn nhất nhỏ hơn

3 n. Khi đó ta có m < √3

n ≤ m + 1. Ta có m ≥ 13 và n chia hết cho lcm{1,2,3,4,5,6,7}. Mặt khác, ta lại có

lcm (m−3, m−2, m−1, m) ≥ m(m−1) (m−2) (m−3) 6

vì chỉ có 2 và 3 là ước chung của bốn số trên.

Từ đó ta có

m(m−1) (m −2) (m−3)

6 ≤ n≤ (m+ 1)3. Suy ra

m ≤ 6

1 + 2

m −1 1 + 3

m−2 1 + 4

m−3

.

Vế trái của bất đẳng thức trên là hàm đơn điệu tăng của biến m, còn vế phải là hàm đơn điệu giảm của biến m. Với m = 13 ta có

13.12.11.10 = 17160 > 6.143

do vậy bất đẳng thức trên không đúng với mọi m ≥ 13.

Vậy không có số m > 420 nào thỏa mãn điều kiện đề bài.

Ví dụ 2.8 (1951 Russian Mathematics Olympiad). Cho các số nguyên dươnga1, a2, ..., an nhỏ hơn 1000và lcm (ai, aj) > 1000với mọi i, j, i 6= j.

Chứng minh rằng

n

X

i=1

1 ai < 2.

(13)

Lời giải. Nếu 1000

m + 1 < a ≤ 1000

m thì có m bội số của a là a,2a, ..., ma không vượt quá1000. Giả sửk1 là số cácainằm trong khoảng

1000

2 ,1000

, k2 là số các ai nằm trong khoảng

1000

3 ,1000 2

,...Như vậy ta có k1 + 2k2 + 3k3 + · · · các số nguyên, không lớn hơn 1000, là bội của ít nhất một số ai nào đó. Vì các bội này là phân biệt (vì lcm (ai, aj) > 1000) nên ta có

k1 + 2k2 + 3k3 +· · · < 1000

⇒ 2k1 + 3k2 + 4k3 +· · · < (k1 + 2k2 + 3k3 +· · ·) + (k1 +k2 + k3 +· · ·)

< 1000 +n < 2000.

Do đó,

n

X

i=1

1

ai ≤ k1 2

1000 +k2 3

1000 +k3 4

1000 +· · · = 2k1 + 3k2 + 4k3 +· · · 1000 < 2.

Ví dụ 2.9. Cho 0< a1 < a2 < ... < an ≤2n là các số nguyên thỏa mãn bội số chung nhỏ nhất của hai số bất kì trong chúng đều lớn hơn 2n.

Chứng minh rằng

a1 >

2n 3

. ( ⌊α⌋ là kí hiệu phần nguyên của số thực α)

Lời giải. Rõ ràng, trong các số trên không tồn tại cặp số nào mà số này chia hết cho số kia (vì nếu trái lại thì bội chung nhỏ nhất của chúng nhỏ hơn hoặc bằng 2n). Ta viết ak = 2tkAk với Ak là số lẻ. Ta thấy các giá trị Ak là phân biệt. Mặt khác từ 1 đến 2n ta có n số lẻ phân biệt.

Do đó các giá trị Ak là các số lẻ từ 1 đến 2n theo một thứ tự nào đó.

Ta xét a1 = 2t1A1. Nếu a1

2n 3

thì 3a1 = 2t13A1 ≤ 2n tức là 3A1 < 2n. Do đó 3A1 là một số lẻ nhỏ hơn 2n, tức là 3A1 = Aj nào đó. Như vậy aj = 2tj3A1. Khi đó [a1, aj] = 2t13A1 = 3a1 ≤ 2n hoặc [a1, aj] = 2tj3A1 = aj ≤ 2n, mâu thuẫn với giả thiết. Vậy điều giả sử là sai, tức là ta có a1 >

2n 3

.

(14)

Ví dụ 2.10. Cho a0 < a1 < a2 < · · · < an là các số nguyên dương.

Chứng minh rằng 1

lcm (a0, a1) + 1

lcm (a1, a2) + · · ·+ 1

lcm (an1, an) ≤ 1− 1

2n. (2.4) Lời giải. Ta chứng minh quy nạp theo n.

Vớin = 1, ta có lcm (a0,a1) ≥ lcm (1,2) = 2. Do đó 1

lcm (a0,a1) ≤ 1−1 2. Giả sử (2.4) đúng với n = k, tức là nếu a0 < a1 < a2 < · · · < ak thì ta

có 1

lcm (a0, a1) + 1

lcm (a1, a2) +· · ·+ 1

lcm (ak1, ak) ≤ 1− 1 2k.

Ta đi chứng minh (2.4) cũng đúng với n = k+ 1. Giả sử a0 < a1 < a2 <

· · · < ak < ak+1 là các số nguyên dương. Ta xét hai trường hợp.

Trường hợp 1. ak+1 ≥ 2k+1. Ta có lcm (ak,ak+1) ≥ ak+1 ≥ 2k+1. Theo giả thiết quy nạp ta có

1

lcm (a0, a1) + 1

lcm (a1, a2) +· · ·+ 1

lcm (ak1, ak) + 1

lcm (ak, ak+1)

≤1− 1

2k + 1

2k+1 = 1− 1 2k+1,

Trường hợp 2. ak+1 < 2k+1. Khi đó ta có 1

lcm (ai1, ai) = gcd (ai1, ai)

ai1ai ≤ ai −ai1

ai1ai = 1

ai1 − 1 ai. Cộng vế các bất đẳng thức như trên với i từ 1 đến k+ 1 ta được

1

lcm (a0, a1) + 1

lcm (a1, a2) +· · ·+ 1

lcm (ak1, ak) + 1

lcm (ak, ak+1)

≤ 1

a0 − 1

ak+1 ≤ 1− 1 2k+1.

Theo phương pháp quy nạp ta được điều phải chứng minh.

Ví dụ 2.11. Cho các số nguyên dương không vượt quá số nguyên dương m không đổi cho trước. Chứng minh rằng nếu mọi số nguyên dương nhỏ hơn hoặc bằng m không chia hết cho cặp hai số nào trong các số được cho ở trên, thì tổng nghịch đảo của các số đã cho nhỏ hơn 3

2.

(15)

Lời giải. Theo giả thiết mọi số nguyên dương nhỏ hơn hoặc bằng m không chia hết cho cặp hai số nào trong các số được cho ở trên nên bội chung nhỏ nhất của hai số bất kì trong các số được cho lớn hơn m. Giả sử cho n số, kí hiệu x1, x2, ..., xn. Với mỗi i, ta có

m xi

bội của xi trong các số 1,2, ..., m, ( ⌊α⌋ là kí hiệu phần nguyên của số thực α). Không có số nào trong chúng là bội của xj với j 6= i vì bội chung nhỏ nhất của xi và xj lớn hơn m. Như vậy ta có

m x1

+

m x2

+· · ·+ m

xn

phần tử đôi một khác nhau của tập hợp {1,2, ..., m} chia hết cho một trong các số x1, x2, ..., xn. Trong các phần tử này không có phần tử nào bằng 1 (trừ trường hợp n = 1, khi đó điều phải chứng minh là hiển nhiên). Do đó

m x1

+

m x2

+ · · ·+ m

xn

≤ m−1.

Với mọi i ta lại có m xi <

m xi

+ 1, suy ra

m 1

x1 + 1

x2 +· · ·+ 1 xn

< m+n−1.

Ta còn phải chứng minh n≤ m+ 1

2 , khi đó ta suy ra được 1

x1 + 1

x2 +...+ 1 xn

< 1 + n−1 m < 3

2.

Thật vậy, ta có các ước số lẻ lớn nhất của các số x1, x2, ..., xn là phân biệt vì nếu trái lại có hai số có cùng ước số lẻ lớn nhất thì một số trong chúng là bội của số còn lại, trái với giả thiết. Suy ra n không vượt quá số các số lẻ trong các số 1,2, ..., m. Do đó n≤ m+ 1

2 .

(16)

3 Liên hệ giữa ước chung lớn nhất và bội chung nhỏ nhất

Định lí 3.1. Cho m, n là hai số nguyên dương. Khi đó ta có

mn = gcd (m, n).lcm (m, n).

Chứng minh. Giả sử m = pα11.pα22...pαkk và n = pβ11.pβ22...pβkk với αi, βi ∈ N, αii ≥ 1, i = 1,2, ..., k. Khi đó ta có

gcd (m, n) =pmin1 {α11}.pmin2 {α22}...pmink {αkk}. lcm (m, n) =pmax1 {α11}.pmax2 {α22}...pmaxk {αkk}. Ta có

gcd (m, n).lcm (m, n) = pmin1 {α11}+max{α11}...pmink {αkk}+max{αkk}

= pα111...pαkkk

= mn.

Nhận xét 3.2. Cho các số nguyên dương a1, a2, ..., an, n ≥ 3 thì đẳng thức

a1a2...an = gcd (a1, a2, ..., an).lcm (a1, a2, ..., an) nói chung là không đúng. Chẳng hạn

gcd (3,3,3).lcm (3,3,3) = 3.3 = 9 < 3.3.3 = 27. Ví dụ 3.3. Cho m, n là các số nguyên dương thỏa mãn

lcm (m, n) + gcd (m, n) = m+n.

Chứng minh rằng một trong hai số đó chia hết cho số còn lại.

Lời giải. Cách 1. Đặt d = gcd (m, n). Khi đó ta có m = ad, n = bd với gcd (a, b) = 1 và

lcm (m, n) = mn

gcd (m, n) = abd.

(17)

Theo bài ra ta có phương trình abd+d = ad+bd hay ab−a−b+ 1 = 0.

Từ đó ta có (a−1) (b−1) = 0, suy ra a = 1 hoặc b = 1. Như vậy ta có m = d, n = bd = bm hoặc n = d, m = an.

Cách 2.Ta cólcm (m, n).gcd (m, n) =m.n, suy ralcm (m, n)vàgcd (m, n) là các nghiệm của phương trình

x2 −(m+n)x+mn = 0.

Dễ thấy hai nghiệm của phương trình là m và n. Do đó {lcm (m, n),gcd (m, n)}= {m, n}. Từ đó ta có điều phải chứng minh.

Ví dụ 3.4 (St. Petersburg 2001). Cho m, n là các số nguyên dương, m > n. Chứng minh rằng

lcm (m, n) + lcm (m+ 1, n+ 1)> 2mn

√m−n. Lời giải. Đặt m = n+k. Khi đó ta có

lcm (m, n) + lcm (m+ 1, n+ 1) = mn

gcd (m, n) + (m+ 1) (n+ 1) gcd (m+ 1, n+ 1)

> mn

gcd (n+k, n) + mn

gcd (m + 1, n+ 1)

= mn

gcd (k, n) + mn

gcd (n+k + 1, n+ 1)

= mn

gcd (k, n) + mn

gcd (k, n+ 1).

Dễ thấygcd (k, n) và gcd (k+ 1, n) không thể có chung ước số nguyên tố nào vì nếu p là một ước số nguyên tố chung của chúng thì p | n và p | n+ 1, điều này là vô lí. Mặt khác gcd (k, n) | k và gcd (k, n+ 1) | k nên ta có gcd (k, n).gcd (k+ 1, n) ≤ k.

Theo bất đẳng thức giữa trung bình cộng và trung bình nhân (AM - GM) ta có

lcm (m, n) + lcm (m+ 1, n+ 1) > mn

gcd (k, n) + mn gcd (k, n+ 1)

≥ 2mn

r 1

gcd (k, n).gcd (k, n+ 1) ≥ 2mn r1

k = 2mn

√m−n.

(18)

Ví dụ 3.5. Cho a, b, c là các số nguyên khác không. Chứng minh rằng (i) [a,(b, c)] = ([a, b],[a, c]),

(ii) (a,[b, c]) = [(a, b),(a, c)]. Lời giải. (i) Ta có

(a, b, c) b

(a, b), c (a, c)

=

b

(a, b) (a, b, c), c

(a, c)(a, b, c)

=

b

(a, b) ((a, b),(b, c)), c

(a, c) ((a, c),(b, c))

=

b, b(b, c) (a, b)

,

c,c(b, c) (a, c)

=

b, b(b, c) (a, b)

, c, c(b, c) (a, c)

=

b, b(b, c) (a, b) , c

,c(b, c) (a, c)

=

(b, c),b(b, c) (a, b)

,c(b, c) (a, c)

=

(b, c),b(b, c)

(a, b) , c(b, c) (a, c)

= (b, c)

1, b

(a, b), c (a, c)

= (b, c). Từ đó suy ra

(b, c) (a, b, c) =

b

(a, b), c (a, c)

. Ta có

(a, b, c) = (a,(b, c)) = a.(b, c) [a,(b, c)]

và b

(a, b) = [a, b]

a ; c

(a, c) = [a, c]

a nên ta được

[a,(b, c)]

a =

[a, b]

a ,[a, c]

a

= 1

a([a, b],[a, c]).

(19)

Vậy [a,(b, c)] = ([a, b],[a, c]). (ii) Từ (i) ta suy ra

a.(b, c)

(a, b, c) = [a, b].[a, c]

[a, b, c]

⇔ abc

(a, b, c) [b, c] = a2bc

(a, b) (a, c) [a, b, c]

⇔ 1

(a, b, c) [b, c] = a

(a, b) (a, c) [a, b, c]

⇔ a[b, c] (a, b, c) = (a, b) (a, c) [a, b, c]

⇔ a[b, c]

[a, b, c] = (a, b) (a, c) (a, b, c)

⇔ (a,[b, c]) = [(a, b),(a, c)]

Ví dụ 3.6. Cho ba số nguyên dương m, n và p. Chứng minh rằng (i) [m, n, p] = mnp(m, n, p)

(m, n) (n, p) (p, m).

(ii) [m, n, p] = (m, n, p) [m, n] [n, p] [p, m]

mnp .

Lời giải.

(i) Cách 1. Suy ra từ ví dụ 3.5 và định lí 3.1.

Cách 2. Đặtd = (m, n, p). Khi đó m = dx, n= dy,p = dz với(x, y, z) = 1.

Ta có

[m, n, p] = [dx, dy, dz] = d[x, y, z], mnp(m, n, p)

(m, n) (n, p) (p, m) = dx.dy.dz.d

d(x, y).d(y, z).d(z, x) = dxyz

(x, y).(y, z).(z, x). Do vậy. điều phải chứng minh tương đương với

[x, y, z] = xyz

(x, y).(y, z).(z, x) trong đó (x, y, z) = 1.

[x, y, z] = [[x, y], z] = [x, y].z ([x, y], z) =

xy (x, y).z

([x, y], z) = xyz

(x, y).([x, y], z). (3.1)

(20)

Từ đó đẳng thức (3.1) tương đương với

([x, y], z) = (x, z).(y, z), (3.2) với điều kiện (x, y, z) = 1.

Đặt q = (x, y). Ta có x = qx1, y = qy1, trong đó (x1, y1) = 1. Hơn nữa do (x, y, z) = 1 nên ta có (q, z) = 1. Do vậy đẳng thức (3.2) tương đương (x1y1, z) = (x1, z).(y1, z). (3.3) Thật vậy, giả sử k = (x1y1, z). Khi đó k |(x1y1) và k | z mà (x1, y1) = 1 nên k | x1 hoặc k | y1. Suy ra k | (x1, z) hoặc k | (y1, z). Do đó k | (x1, z).(y1, z).

Ngược lại nếu l = (x1, z) thì l | x1 và l |z, suy ral | x1y1 nên l | (x1y1, z).

Tương tự (x1, z) | (x1y1, z). Do đó (3.3) được chứng minh. Từ đó ta có (i) được chứng minh hoàn toàn.

(ii) Suy ra từ (i) bằng cách sử dụng định lí 3.1.

Ví dụ 3.7 (USAMO 1972). Cho các số nguyên a, b và c. Chứng minh rằng

[a, b, c]2

[a, b] [b, c] [c, a] = (a, b, c)2 (a, b) (b, c) (c, a). Lời giải.

Cách 1. Áp dụng Ví dụ 3.6 ta có [a, b, c] = abc(a, b, c)

(a, b) (b, c) (c, a) = (a, b, c) [a, b] [b, c] [c, a]

abc .

Do đó

[a, b, c]2 = abc(a, b, c)

(a, b) (b, c) (c, a).(a, b, c) [a, b] [b, c] [c, a]

abc = (a, b, c)2[a, b] [b, c] [c, a]

(a, b) (b, c) (c, a) . Từ đó ta có điều phải chứng minh.

Cách 2. Giả sử a = pα11· · ·pαnn, b = pβ11· · ·pβnn, c = pγ11· · ·pγnn, ở đó p1, p2, ..., pnlà các số nguyên tố phân biệt, vàα1, ..., αn1, ..., βn1, ..., γn

(21)

là các số nguyên không âm. Khi đó ta có

[a, b, c]2

[a, b] [b, c] [c, a] =

n

Q

i=1

p2 maxi {αiii}

n

Q

i=1

pmaxi {αii}

n

Q

i=1

pmaxi {βii}

n

Q

i=1

pmaxi {γii}

=

n

Y

i=1

p2 maxi {αiii}−max{αii}−max{βii}−max{γii}

(a, b, c)2

(a, b) (b, c) (c, a) =

n

Q

i=1

p2 mini {αiii}

n

Q

i=1

pmini {αii}

n

Q

i=1

pmini {βii}

n

Q

i=1

pmini {γii}

=

n

Y

i=1

p2 mini {αiii}−min{αii}−min{βii}−min{γii}

Ta cần chứng minh với các số nguyên không âm α, β, γ bất kì 2 max{α, β, γ} −max{α, β} −max{β, γ} −max{γ, α}

= 2 min{α, β, γ} −min{α, β} −min{β, γ} −min{γ, α}. (3.4) Thật vậy, do tính đối xứng, không mất tổng quát ta giả sử α ≤ β ≤ γ.

Khi đó ta có (3.4) tương đương với

2γ −β −γ −γ = 2α−α−β−α. (3.5) Đẳng thức (3.5) hiển nhiên đúng. Từ đó ta có điều phải chứng minh.

Chú ý: Ta thấy phương pháp này có thể dùng để chứng minh được các ví dụ 3.5, ví dụ 3.6 theo cách tương tự.

Nhận xét 3.8. Từ ví dụ trên ta có [a, b] [b, c] [c, a]

[a, b, c]2 và (a, b) (b, c) (c, a) (a, b, c)2 là các số nguyên bằng nhau.

Ví dụ 3.9. Cho n ≥ 2 số nguyên dương a1, a2, ..., an. Chứng minh rằng a1a2...an ≥[a1, a2, ..., an] (a1, a2, ..., an).

(22)

Lời giải. Đặt

A = a1a2...an (a1, a2, ..., an). Với mỗi i = 1, n ta có

A = ai.a1...ai1.ai+1...an (a1, a2, ..., an) ...ai

Từ đó suy ra

A...[a1, a2, ..., an]. Do đó

A ≥[a1, a2, ..., an].

4 Bài tập đề nghị

Bài toán 4.1. Tìm các số tự nhiêna, b thỏa mãn(a, b) = 12,[a, b] = 532.

Bài toán 4.2. Tìm hai số a, b nguyên dương thỏa mãn a2+b2 = 85113, [a, b] = 1764.

Bài toán 4.3. Chứng minh rằng (a, b)n = (an, bn) với mọi số tự nhiên n.

Bài toán 4.4. Cho các số nguyên a, b, c khác không. Chứng minh rằng ([a, b],[b, c],[c, a]) = [(a, b),(b, c),(c, a)].

Bài toán 4.5. Chứng minh rằng gcd

n−1 k −1

,

n k + 1

,

n+ 1 k

= gcd

n−1 k

,

n+ 1 k+ 1

,

n k−1

Bài toán 4.6 (USAMO 1993). Cho a, b là các số nguyên dương lẻ. Dãy số (fn) được xác định như sau f1 = a, f2 = b, với n ≥3 thì fn là ước số lẻ lớn nhất của fn1+fn2. Chứng minh rằng fn không đổi với n đủ lớn và xác định giá trị đó theo a và b.

(23)

Bài toán 4.7 (1997 Canadian Mathematical Olympiad). Có bao nhiêu cặp số nguyên dương (x, y) với x ≤ y thỏa mãn gcd (x, y) = 5! và lcm (x, y) = 50!.

Bài toán 4.8 (1995 Russian Mathematical Olympiad). Dãy các số tự nhiên a1, a2, ... thỏa mãn điều kiện gcd (ai, aj) = gcd (i, j) với mọi i 6= j.

Chứng minh rằng ai = i với mọi i.

Bài toán 4.9 (AIME 1985). Cho dãy số (an) được xác định bởi an = 100 +n2, n = 1,2, ... Với mỗi n, đặt dn = (an, an+1). Tìm max

n1 dn. Tài liệu tham khảo

[1 ] Hà Huy Khoái, Chuyên đề bồi dưỡng học sinh giỏi toán trung học phổ thông Số học, NXB Giáo dục, 2006.

[2 ] Đặng Hùng Thắng - Nguyễn Văn Ngọc - Vũ Kim Thủy, Bài giảng số học, NXB Giáo dục Việt Nam, 2010.

[3 ] Dương Quốc Việt (chủ biên) - Đàm Văn Nhỉ, Cơ sở lí thuyết số và đa thức, NXB Đại học Sư phạm, 2012.

[4 ] Dương Quốc Việt (chủ biên), Nguyễn Đạt Đăng, Lê Văn Đính, Lê Thị Hà, Đặng Đình Hanh, Đào Ngọc Minh, Trương Thị Hồng Thanh, Phan Thị Thủy, Bài tập cơ sở lí thuyết số và đa thức, NXB Đại học Đại học Sư phạm, 2014.

[5 ] Titu Andreescu, Dorin Andrica, Zuming Feng, 104 number theory problems, Birkhauser Boston - Basel - Berlin, 2006.

[6 ] David A. Santos, Elementary Number Theory Notes C, 2004.

[7 ] Titu Andreescu, Dorin Andrica, Number Theory Structures, Ex- ample, and Problems, Birkhauser.

Tài liệu tham khảo

Tài liệu liên quan

Nhóm giáo viên Toán tiếp sức Chinh phục kì thi THPT năm 2020 Trong các đề thi thử và đề thi minh họa của BGD&amp;ĐT, các em học sinh gặp nhiều bài toán giá trị lớn nhất

Người ta cắt ở bốn góc của tấm nhôm đó bốn hình vuông bằng nhau, mỗi hình vuông có cạnh bằng x (cm), rồi gập tấm nhôm lại như hình vẽ dưới đây để được một cái hộp không

Định lý Ptoleme hay đẳng thức Ptoleme là một đẳng thức trong hình học Euclid miêu tả quan hệ giữa độ dài bốn cạnh và hai đường chéo của một tứ giác nội tiếp.. Định lý

Trong đề tham khảo của Bộ GD lần 1 và lần 2, cũng như đề thi thử của các sở giáo dục, các trường phổ thông năm 2020 thường có bài toán liên quan đến GTLN-GTNN của hàm

DẠNG TOÁN: Đây là dạng toán max, min của hàm trị tuyệt đối có chứa tham số.. GTLN - GTNN CỦA HÀM TRỊ TUYỆT ĐỐI CÓ CHỨA

Hỏi chiều rộng nhỏ nhất của đoạn đường đầu tiên gần nhất với giá trị nào trong các giá trị bên dưới để ô tô có thể đi vào GARA được.. (giả thiết ô tô không đi ra

Câu 38: Trên bàn có một cố nước hình trụ chứa đầy nước, có chiều cao bằng 3 lần đường kính của đáy;.. Một viên bi và một khối nón đều

Câu 2: Kết quả của phép tính nào sau đây là số nguyên tố... Chọn phát biểu đúng trong các phát